Difference between revisions of "1971 AHSME Problems/Problem 5"

(added second solution)
 
(2 intermediate revisions by 2 users not shown)
Line 31: Line 31:
  
  
== Solution ==  
+
== Solution 1 ==  
 +
We see that the measure of <math>P</math> equals <math>(\widehat{BD}-\widehat{AC})/2</math>, and that the measure of <math>Q</math> equals <math>\widehat{AC}/2</math>.
 +
Since <math>\widehat{BD} = \widehat{BQ} + \widehat{QD} = 42^{\circ} + 38^{\circ} = 80^{\circ}</math>, the sum of the measures of <math>P</math> and <math>Q</math> is <math>\widehat{BD}/2 = 80^{\circ}/2 = 40^{\circ} \Longrightarrow \boxed{\textbf{(C) }40^{\circ}}</math>.
  
We see that the measure of <math>P</math> equals <math>(\widehat{BD}-\widehat{AC})/2</math>, and that the measure of <math>Q</math> equals <math>\widehat{AC} \div 2</math>.
+
== Solution 2 ==
Since <math>\widehat{BD} = \widehat{BQ} + \widehat{QD} = 42^{\circ} + 38^{\circ} = 80^{\circ}</math>, the sum of the measures of <math>P</math> and <math>Q</math> is $\widehat{BD} \div 2 = 80^{\circ} \div 2 = 40^{\circ} \Longrightarrow \textbf{(C) }.
+
Arcs are measured by the angle measures of their corresponding [[central angle|central angles]]. Thus, the [[inscribed angle]] <math>\measuredangle BAQ = \tfrac{42^{\circ}}2 = 21^{\circ}</math>, and, likewise, <math>\measuredangle QCD = \tfrac{38^{\circ}}2 = 19^{\circ}</math>. Thus, by [[supplementary]] angles, <math>\measuredangle PAQ = 180^{\circ} - \measuredangle BAQ = 159^{\circ}</math>, and <math>\measuredangle PCQ = 180^{\circ} - \measuredangle QCD = 161^{\circ}</math>. Because the sum of the [[interior angle]] measures of a quadrilateral add to <math>360^{\circ}</math>, we see that <math>\measuredangle P + \measuredangle Q = 360^{\circ} - \measuredangle PAQ - \measuredangle PCQ = 360^{\circ} - 159^{\circ} - 161^{\circ} = 40^{\circ}</math>. Thus, our answer is <math>\boxed{\textbf{(C) }40^{\circ}}</math>.
 +
 
 +
== See Also ==
 +
{{AHSME 35p box|year=1971|num-b=4|num-a=6}}
 +
{{MAA Notice}}
 +
[[Category:Introductory Geometry Problems]]

Latest revision as of 10:03, 1 August 2024

Problem 5

Points $A,B,Q,D$, and $C$ lie on the circle shown and the measures of arcs $\widehat{BQ}$ and $\widehat{QD}$ are $42^\circ$ and $38^\circ$ respectively. The sum of the measures of angles $P$ and $Q$ is

$\textbf{(A) }80^\circ\qquad \textbf{(B) }62^\circ\qquad \textbf{(C) }40^\circ\qquad \textbf{(D) }46^\circ\qquad  \textbf{(E) }\text{None of these}$

[asy] size(3inch); draw(Circle((1,0),1)); pair A, B, C, D, P, Q; P = (-2,0); B=(sqrt(2)/2+1,sqrt(2)/2); D=(sqrt(2)/2+1,-sqrt(2)/2); Q = (2,0); A = intersectionpoints(Circle((1,0),1),B--P)[1]; C = intersectionpoints(Circle((1,0),1),D--P)[0]; draw(B--P--D); draw(A--Q--C); label("$A$",A,NW); label("$B$",B,NE); label("$C$",C,SW); label("$D$",D,SE); label("$P$",P,W); label("$Q$",Q,E); //Credit to chezbgone2 for the diagram[/asy]


Solution 1

We see that the measure of $P$ equals $(\widehat{BD}-\widehat{AC})/2$, and that the measure of $Q$ equals $\widehat{AC}/2$. Since $\widehat{BD} = \widehat{BQ} + \widehat{QD} = 42^{\circ} + 38^{\circ} = 80^{\circ}$, the sum of the measures of $P$ and $Q$ is $\widehat{BD}/2 = 80^{\circ}/2 = 40^{\circ} \Longrightarrow \boxed{\textbf{(C) }40^{\circ}}$.

Solution 2

Arcs are measured by the angle measures of their corresponding central angles. Thus, the inscribed angle $\measuredangle BAQ = \tfrac{42^{\circ}}2 = 21^{\circ}$, and, likewise, $\measuredangle QCD = \tfrac{38^{\circ}}2 = 19^{\circ}$. Thus, by supplementary angles, $\measuredangle PAQ = 180^{\circ} - \measuredangle BAQ = 159^{\circ}$, and $\measuredangle PCQ = 180^{\circ} - \measuredangle QCD = 161^{\circ}$. Because the sum of the interior angle measures of a quadrilateral add to $360^{\circ}$, we see that $\measuredangle P + \measuredangle Q = 360^{\circ} - \measuredangle PAQ - \measuredangle PCQ = 360^{\circ} - 159^{\circ} - 161^{\circ} = 40^{\circ}$. Thus, our answer is $\boxed{\textbf{(C) }40^{\circ}}$.

See Also

1971 AHSC (ProblemsAnswer KeyResources)
Preceded by
Problem 4
Followed by
Problem 6
1 2 3 4 5 6 7 8 9 10 11 12 13 14 15 16 17 18 19 20 21 22 23 24 25 26 27 28 29 30 31 32 33 34 35
All AHSME Problems and Solutions

The problems on this page are copyrighted by the Mathematical Association of America's American Mathematics Competitions. AMC logo.png